LSAT and Law School Admissions Forum

Get expert LSAT preparation and law school admissions advice from PowerScore Test Preparation.

 Administrator
PowerScore Staff
  • PowerScore Staff
  • Posts: 8916
  • Joined: Feb 02, 2011
|
#63960
Complete Question Explanation

Weaken. The correct answer choice is (A)

The doctor quoted here makes a questionable argument: many alternative medicine-used herbs are
safe to consume, so they should always be allowed as prescribed remedies for serious illnesses,
because they won’t harm patients, and might help. We should note the doctor’s use of strong
language: the assertion is that alternative herb prescribed remedies should always be allowed.

The stimulus is followed by a weaken question, which means that we should look for the answer
choice which weakens the doctor’s conclusion. The correct answer choice will likely provide either a
previously unstated detriment that goes along with the use of such herbs, or a benefit associated with
their avoidance.

Answer choice (A): This is the correct answer choice, providing a previously unstated detriment
associated with the use of such alternative medicine herbs. If the use of such herbs comes at the cost
of effective conventional medicine, then patients might indeed be harmed (by not having access to
conventional methods that have been proven effective).

Answer choice (B): This answer choice does not weaken the doctor’s argument, which allows for the
possibility that some are not effective, but concludes that their prescription should be allowed if they
do no harm.

Answer choice (C): The doctor discusses herbs that have been proven safe to consume, so the fact
that some people are allergic to some medicines does not weaken the doctor’s argument, and this
answer choice is incorrect.

Answer choice (D): The motivations of alternative medicine purveyors does not affect the strength of
the doctor’s conclusion, which is that such herbs should be allowed to be prescribed when they do no
harm and may do some good.

Answer choice (E): This answer choice would not weaken the author’s argument—it may strengthen
the conclusion, providing a benefit of such herbs and making it more likely that they might help.
 anahi78
  • Posts: 16
  • Joined: Nov 09, 2013
|
#12441
The correct answer is A however it seems that it is strengthening the doctor's argument, not weakening. Please explain how this is the correct choice? Thanks! Anahi
 Ron Gore
PowerScore Staff
  • PowerScore Staff
  • Posts: 220
  • Joined: May 15, 2013
|
#12452
Hi, Anahi! Thanks for your question.

As you point out, since this is a Weaken question, the correct answer choice will undermine the conclusion, while the incorrect answer choices will have no effect on the conclusion or could strengthen it.

Your ask why answer choice (A) is the correct choice, when it seems to strengthen the conclusion. Let's start by looking at the argument.

The conclusion is that advocates of certain herbal remedies for serious illnesses should always be allowed to prescribe them. The reasons given for this are: 1) their patients will not be harmed by the herbs; and 2) the patients may be helped by the use of the products.

Your prephrase is that the correct answer choice will provide some reason to think that the herbal remedies should not always be allowed to prescribe them, even if the reasons provided in the stimulus are factually accurate.

Answer choice (A) provides such a reason. The key word in this answer choice is "effective." The impact of this answer choice is that, while the herbal remedies may not directly harm the patients, many practitioners and patients neglect "more effective" conventional remedies in favor of herbal remedies. The concern being that if practitioners are always permitted to prescribe herbal remedies, for which there is little evidence of effectiveness, then they may be more likely to neglect more effective conventional remedies.

Thanks!

Ron
 reop6780
  • Posts: 265
  • Joined: Jul 27, 2013
|
#16571
The answer is A correct while I did not really find any appealing answer.

I seem to miss specifically "most weaken" questions repeatedly.

I read the LR bible again, and the book does not distinguish "most weaken" from any other "weaken" questions.

Recently, I got "most weaken" question wrong again, and one of powerscore instructors gave me an answer that "most weaken" question allows the answer to be correct only when the answer is the link between premises and conclusion.

I thought this would be the reason why I keep getting "most weaken" questions wrong since I find other weaken questions easy by spotting an answer that attacks the conclusion.

Is there different strategy to attack "most weaken" questions?
 David Boyle
PowerScore Staff
  • PowerScore Staff
  • Posts: 836
  • Joined: Jun 07, 2013
|
#16586
reop6780 wrote:The answer is A correct while I did not really find any appealing answer.

I seem to miss specifically "most weaken" questions repeatedly.

I read the LR bible again, and the book does not distinguish "most weaken" from any other "weaken" questions.

Recently, I got "most weaken" question wrong again, and one of powerscore instructors gave me an answer that "most weaken" question allows the answer to be correct only when the answer is the link between premises and conclusion.

I thought this would be the reason why I keep getting "most weaken" questions wrong since I find other weaken questions easy by spotting an answer that attacks the conclusion.

Is there different strategy to attack "most weaken" questions?
Hello reop6780,

Not always. --Law Services doesn't like to be sued, one hears, so that just in case they pick two answers that both weaken, they sometimes say "most weaken" so that they can't be sued by some test taker who screams "Hey! There's more than one right answer!"
Just see which answer most weakens, that's all. If one answer could maybe weaken a tiny little bit, and another would weaken hugely, pick the second one.

As for question 5: answer A is good because even if herbal remedies are harmless, answer A shows that those remedies *displace* better, conventional remedies. Obviously, this could hurt the patient.

Hope this helps,
David
 reop6780
  • Posts: 265
  • Joined: Jul 27, 2013
|
#16614
Thank you for clarifying, David :lol:
 Rita
  • Posts: 38
  • Joined: Sep 30, 2016
|
#29505
Hi,

Why is answer choice C incorrect? Even if the drugs were proven safe in clinical trials, C suggests some patients may still have allergic reactions. Thus, the premise that "patients will not be harmed" is weakened.

Thanks,
Rita
 Adam Tyson
PowerScore Staff
  • PowerScore Staff
  • Posts: 5153
  • Joined: Apr 14, 2011
|
#29507
Consider all the evidence in the stimulus, Rita, and you'll see that the herbs in question have been proven to be safe. While some folks may have allergic reactions to some medicines, that doesn't mean anyone will have such reactions to these particular safe herbs, does it?,

Even if some folks do have such reactions Answer A is so much more harmful, and we need to pick the best answer of the bunch, the one that weakens the most (as opposed to one that may just weaken some, like C might do). Focus on that comparison between answers, and never forget what the instructions and the question stem tell you to do.
 Rita
  • Posts: 38
  • Joined: Sep 30, 2016
|
#29519
Thanks Adam!
 hskhader
  • Posts: 6
  • Joined: May 07, 2018
|
#45770
Hello,
I have a couple of questions regarding this specific question

For starters, despite the answers given, is it possible that we can attack the question based on the gap between the premises themselves? Specifically.
1) MANY (not all) herbs have been proven safe to consume
and
2) Since patients will NOT be harmed

Is one way of attacking this question is by stating something along the lines " although many have been proven safe to consume, some may still be unsafe.

Thank You

Get the most out of your LSAT Prep Plus subscription.

Analyze and track your performance with our Testing and Analytics Package.